Inscription / Connexion Nouveau Sujet

1 2 3 +


Posté par
jb2017
re : Inequation 07-08-17 à 12:17

Rebonjour
Néanmoins f est convexe sur  \Omega  implique bien que f est convexe sur \Delta et cela est une information importante.

Posté par
jb2017
re : Inequation 07-08-17 à 13:04

Rebonjour encore
Maintenant je ne suis pas convaincu que f est concave sur \Omega
En effet si on  considère  g, l'application de
\Omega'=]0,\infty[^2 vers \R  
définie par (a,b) \rightarrow a(a^{1/3}+b^{1/3})
puis je considère deux exemples où  h(t)=f( M(t)) =f(t A +(1- t) B )  
avec A=(1,2) et B=(1,3)  et
puis A=(1,2) et B=(2,2)    
je trouve dans un cas h''(t)>0 et dans l'autre h''(t)<0.

Posté par
jb2017
re : Inequation 08-08-17 à 00:16

Rebonjour
Finalement j'ai mis ma solution sur le site où j'ai trouvé le problème

Je verrai bien si on trouve des erreurs.
J'ai mis un peu de Lagrange multiplicateurs mais pas trop.

Posté par
etniopal
re : Inequation 08-08-17 à 00:31


Ce n'est pas  (x,y) \rightarrow x(x^a+y^a) qui est convexe .
C'est (x,y) \rightarrow x/(x^a+y^a)  est donc aussi (x,y,z) \rightarrow x/(x^a+y^a) .

Posté par
jb2017
re : Inequation 08-08-17 à 01:06

Oui c'est bien de f(x,y)= x/(x^a+y^a) dont je parle.
Le contre-exemple est avec A=(1,2) et B(=2,2) et a=1/3
M(t)=t* A +(1-t) B  donne $h(t)= \frac{2-t}{2^{1/3}+(2-t)^{1/3}}$
Graphiquement on voit que h''(t)<0,\;  \forall t entre  0 et 1.  

Posté par
etniopal
re : Inequation 08-08-17 à 10:33

J'ai trouvé mon erreur !

Le produit de 2 applications convexes > 0  n'est pas toujours convexe .
J'aurais pu penser bêtement  à x  x  et x 1/x

Posté par
jb2017
re : Inequation 08-08-17 à 13:31

Oui. Je crois que l'on est d'accord que la fonction n'est pas convexe sur $\Omega$ mais  c'est "sûr" que la fonction est convexe sur \Delta mais le démontrer reste je pense un problème ouvert.
Moi aussi j'ai voulu passer par là mais cela me semble bien compliqué.
Je pense finalement avoir trouvé la démonstration de l'inégalité que j'ai légèrement améliorée pour ne pas utiliser du tout Lagrange.   Je pense que c'est ce Forum qui m'a motivé a chercher alors que je suis en vacances.


Posté par
Alexique
re : Inequation 09-08-17 à 11:26

Citation :
Je pense finalement avoir trouvé la démonstration de l'inégalité que j'ai légèrement améliorée pour ne pas utiliser du tout Lagrange

Nous sommes tout ouïe

Posté par
Sylvieg Moderateur
re : Inequation 11-08-17 à 18:11

Bonjour,
J'ai regardé la démonstration de jb2017 dans

Après avoir passé un temps conséquent sur l'étape 2, voici plusieurs remarques :
Young n'est pas nécessaire. Il suffit de multiplier x 1/2 + (1/2)x2 par a3 .
Dans l'itération de l'étape 2, il me semble que l'exposant de x n'est pas 2n mais 2n .
On peut simplifier par a3 l'inégalité obtenue par cette itération :
x 1 - 1/(2n) + x2n/2n
Et cette dernière inégalité n'est pas toujours vraie pour x positif. Regarder n= 3 par exemple.

Il se peut que j'ai mal compris l'itération

Posté par
alainpaul
re : Inequation 15-08-17 à 12:11

Bonjour,

Nous avons:a,b,c \in R^{*+} , \frac{a^3}{a+b}+\frac{b^3}{b+c}+\frac{c^3}{c+a} \geq 3\frac{a^3+b^3+c^3}{2(a+b+c)}
c'est-à-dire: \frac{a^3}{a+b}+\frac{b^3}{b+c}+\frac{c^3}{c+a} \geq \frac{3}{2(a+b+c)}

Nous devons donc maximiser (a+b+c)  sous la contrainte (a^3+b^3+c^3=3)

Ai-je bien compris le problème?

Alain

Posté par
alainpaul
re : Inequation 15-08-17 à 12:57

Bon,


Erreur de frappe,lire 2 ème ligne  9/2


Alain

Posté par
DOMOREA
re : Inequation 15-08-17 à 14:57

bonjour,
@etniopal

Citation :
Le produit de 2  fonctions strictement convexes > 0 étant aussi strictement  convexe  
  ceci est faux un exemple simple:
f(x)=x^2; g(x)= e^{-x}    fg n'est pas convexe sur l'intervalle où x^2-4x+2 est négatif . calcul de la dérivée seconde....  il faudrait comme condition suffisante  mais non nécessaire que les deux fonctions convexes et positives soient en plus toutes deux croissantes ou toutes deux décroissantes

Posté par
DOMOREA
re : Inequation 15-08-17 à 15:10

re
excuse à etniopal, je n'avais pas vu que tu avais corrigé

Posté par
etniopal
re : Inequation 15-08-17 à 15:32

DOMOREA
Il y a presqu'un mois , tu affirmais que " g est une fonction convexe comme produit de 2 fonctions convexes positives  " .
Comme disais Coluche , tu m'as enduit d'erreur !

Pas grave !

Posté par
Sylvieg Moderateur
re : Inequation 15-08-17 à 18:24

Bonjour,
@alainpaul, je ne vois pas d'où viennent le 3 et le 2 dans la première ligne.

Ce n'est pas plutôt  \frac{a^3}{a+b}+\frac{b^3}{b+c}+\frac{c^3}{c+a} \geq \frac{a^3+b^3+c^3}{a+b+c} ?

Qui donne  \frac{a^3}{a+b}+\frac{b^3}{b+c}+\frac{c^3}{c+a} \geq \frac{3}{a+b+c}

Le problème, c'est que a+b+c peut être supérieur à 2.

Par contre, si ton 9/2 de la seconde ligne peut se démontrer, c'est gagné car Rasez a démontré a+b+c 3 .

Posté par
DOMOREA
re : Inequation 15-08-17 à 18:40

bonjour,
non  sylvieg ; l'inégalité donnée par alainpaul est correcte mais ....
alainpaul ,  ta minoration est beaucoup trop grossière pour conduire au résultat

Posté par
Sylvieg Moderateur
re : Inequation 15-08-17 à 18:56

La première inégalité  \frac{a^3}{a+b}+\frac{b^3}{b+c}+\frac{c^3}{c+a} \geq 3\frac{a^3+b^3+c^3}{2(a+b+c)} est correcte ?
Je ne vois pas d'où elle vient

Posté par
DOMOREA
re : Inequation 15-08-17 à 19:35

re,
@sylvieg
le "3" du post de alainpaul ( celui qui est écrit en gros) est sans doute une coquille

Posté par
alainpaul
re : Inequation 16-08-17 à 10:04

Bonjour,

La formule proposée correspond  au cas où l'on suppose les 3 rapports  égaux,
le signa égal alors s'impose.

A creuser,

Alain

Posté par
alainpaul
re : Inequation 17-08-17 à 10:14

Bonjour,


Suis-je assez clair?


Alain

Posté par
Sylvieg Moderateur
re : Inequation 18-08-17 à 09:51

Bonjour,
@alainpaul
Pas vraiment

Sinon, suite à l'absence de réaction à mon message du 11/08 vers 18h, j'ai fini par éditer la démarche de jb2017 dans son étape 2 du site
Mais mon anglais catastrophique me handicape sérieusement

Posté par
alainpaul
re : Inequation 18-08-17 à 10:39

Bonjour,

En appliquant la règle concernant la somme de rapports égaux:

\frac{n_1}{d_1}=\frac{n_2}{d_1}=\frac{n_3}{d_3}=\frac{n_1+n_2+n_3}{d_1+d_2+d_3}
nous pouvons écrire: S=3\times \frac{n_1+n_2+n_3}{d_1+d_2+d_3}

May I help you?

Alain

Posté par
Sylvieg Moderateur
re : Inequation 18-08-17 à 11:32

Why not next time

Posté par
jb2017
re : Inequation 23-08-17 à 22:22

Bonjour
Je réponds au message du Sylvieg du  11-08-17 à 18:1 concernant la solution que j'ai donnée sur le site .
D'abord @Sylvieg je ne comprends pas ta phrase
"Dans l'itération de l'étape 2, il me semble que l'exposant de  x  n'est pas  2n  mais  2^n "
parce que j'ai bien écris x^{2^n} (c'est à dire l'exposant de x est 2^n) donc je ne vois pas le problème. L'inégalité que j'ai démontrée (que l'on peut vérifier par une simple récurrence) et bien valable pour tout x positif et tout n.
Maintenant concernant le facteur  a^3 on peut effectivement le multiplier à la fin (ce que je n'ai pas fait) mais cela ne change pas fondamentalement la démonstration car l'inégalité avec ou sans  facteur a^3, c'est bien l'utilisation de Young que j'utilise par récurrence.

Posté par
Sylvieg Moderateur
re : Inequation 24-08-17 à 08:07

Bonjour,
@jb2017,
J'ai modifié les exposants le 17 août. On peut le voir en cliquant sur le "edited" correspondant.
J'aurais préféré utiliser "add a comment", mais je n'ai pas le droit.
C'est un peu paradoxal de ne pas avoir le droit de commenter alors qu'on a le droit de modifier, sous réserve d'acceptation cependant.
Je me suis mal exprimée dans mon message sur l'île du 11/08. C'est l'inégalité avec l'exposant 2n qui est fausse pour n = 3 .
Celle avec 2n est bonne, mais ne permet pas de trouver un résultat intéressant quand on passe à la limite.
D'où le "No" que j'ai ajouté à la fin de ce "step".

En effet, \frac{a^{2^{n}}}{2^{n}} a une limite infinie si a> 1.

Posté par
Sylvieg Moderateur
re : Inequation 24-08-17 à 08:42

Voir aussi le message de YvesM dans quand il signale l'absence de a3 il y a deux semaines.

Sinon, la seconde démonstration dans le cas max(a, b, c) > 2 est super.
Le cas a2+b2+c2 < 2 est traité par YvesM dans il y a 8 jours.
Le cas a+b+c < 2 est évident.
Reste à trouver pour a, b, c tous les trois inférieurs à 2 , avec a2+b2+c2 > 2 et a+b+c > 2

Posté par
alainpaul
re : Inequation 24-08-17 à 10:01

Bonjour,

Pour l'énoncé initial quel est la valeur d' .

Pour ce minimum, les variables ne devraient-elles jouer un rôle identique?


Alain

Posté par
Sylvieg Moderateur
re : Inequation 24-08-17 à 10:35

Bon jeudi alainpaul,
Dans l'énoncé initial la valeur de est bien 3/2 . Voir
Nous sommes tous persuadés que le minimum est atteint pour a=b=c=1 .
Mais la démonstration ressemble à une anguille...

Posté par
alainpaul
re : Inequation 24-08-17 à 11:47

Well,

Merci,tu confirmes ce que je pensais.

L'anglaise  démonstration compliquée me semble  fait appel à plusieurs
théorèmes .

Alain

Posté par
jb2017
re : Inequation 24-08-17 à 13:18

Bonjour
@Sylvieg
Effectivement je commence à comprendre.
Si je n'ai pas réagi c'est parce que j'étais en vacances. C'est vrai que ce n'est pas normal que l'on puisse modifier un texte lorsqu'on n'est pas l'auteur. De ta part cela ne me gêne pas  mais il y a un "malveillant" qui pourrait faire des dégâts.
En tout cas merci pour ta lecture attentive.
Dès que je retrouverai un peu de temps je vais essayer de lire ce qu'à fait Y. M et peut être on finira par trouver dans ce cas une solution collective.

Posté par
Sylvieg Moderateur
re : Inequation 24-08-17 à 16:09

Merci pour ta réaction mesurée à mon intervention qui pouvait sembler indélicate.
En fait je trouvais dommage que les recherches s'arrêtent alors qu'une solution complète n'était pas trouvée.
Pour les modifications, il y a quand même un garde fou : Elles n'apparaissent qu'après validation par un ou des responsables du site.
Jai voulu en faire une autre ensuite, pour tenter d'adoucir et clarifier mon brutal et peu clair « No » ; elle a été refusée.

Ma très modeste pierre à l'édifice : Avec ce que YvesM a trouvé sur l'autre site, le cas a²+b²+b² 4,5 est traité.
C'est un tout petit peu mieux que a²+b²+b² 2 .

Posté par
alainpaul
re : Inequation 25-08-17 à 10:41

Bonjour,

Je me demande si nous ne tenons pas la solution;je vois 2 possibilités:

1- les  3 rapports figurant dans l'expression du minimum sont égaux ,cela
nous conduit à \alpha = min( \frac{9}{2(a+b+c)})=\frac{3}{2},

2- les rapports vérifient:\frac{a^3}{a+b}\geq \frac{b^3}{b+c}\geq \frac{c^3}{c+a}

soit:\frac{a^3}{a+b}=\lambda \frac{b^3}{b+c}=\gamma \frac{c^3}{c+a} ;\lambda,\gamma \geq 1   et encore:3 \frac{a^3+\lambda b^3+\gamma c^3}{2(a+b+c)}\geq \frac{9}{2(a+b+c)}

D'où \alpha =\frac{3}{2}

Est-ce correct?

Alain

Posté par
Sylvieg Moderateur
re : Inequation 26-08-17 à 08:48

Bonjour,
@alainpaul :
Si j'ai bien compris, tu utilises

Si \frac{n_1}{d_1} = \frac{n_2}{d_1} = \frac{n_3}{d_3}  alors \frac{n_1}{d_1} = \frac{n_2}{d_1} = \frac{n_3}{d_3} = \frac{n_1+n_2+n_3}{d_1+d_2+d_3}

D'où \frac{n_1}{d_1}+\frac{n_2}{d_1}+\frac{n_3}{d_3} = 3\times \frac{n_1+n_2+n_3}{d_1+d_2+d_3}

D'accord pour 1- en utilisant a+b+c 3 .

Par contre, pour 2- , je n'arrive pas à trouver quelque chose sur \frac{a^3}{a+b}+ \frac{b^3}{b+c}+\frac{c^3}{c+a} .
et ne veulent pas disparaître

Posté par
alainpaul
re : Inequation 26-08-17 à 11:37

Bonjour Sylvieg,

" et, ne veulent pas disparaître"

pour 2-  N'avons nous pas  une majoration de l'expression 1-    ?


Alain

Posté par
Sylvieg Moderateur
re : Inequation 26-08-17 à 11:40

Posté par
alainpaul
re : Inequation 27-08-17 à 10:18

Bon dimanche,

Y-aurait-il un 'blame' dans ce que j'ai écrit?

2 ème cas les trois rapports ne sont pas égaux alors :
un des deux coefficients est strictement supérieur  à 1 et:

a^3+\lambda b^3+ \gamma c^3 \>a^3+b^3+c^3

cela nous conduit à retenir pour minimum  celui donné par  (1).

Ton avis,


Alain

Posté par
alainpaul
re : Inequation 27-08-17 à 10:19

Oups,

le signe  >   a disparu!

Posté par
Sylvieg Moderateur
re : Inequation 27-08-17 à 14:35

Dans la situation 2- , on a :

\frac{a^3}{a+b}+ \frac{\lambda b^3}{b+c}+\frac{\gamma c^3}{c+a} \geq \frac{a^3}{a+b}+ \frac{b^3}{b+c}+\frac{c^3}{c+a} .

Je ne vois pas comment en déduire \frac{a^3}{a+b}+ \frac{b^3}{b+c}+\frac{c^3}{c+a} \geq \frac{3}{2} .

Posté par
alainpaul
re : Inequation 28-08-17 à 16:58

Bonsoir,

Es-tu d'accord que:

3\frac{a^3+\lambda b^3+\gamma c^3}{2(a+b+c)} >3\frac{a^3+ b^3+ c^3}{2(a+b+c)}
cela nous permet de retenir le minimum calculé en  1)


Alain

Posté par
Sylvieg Moderateur
re : Inequation 28-08-17 à 17:21

Oui, mais comment justifier

\frac{a^3}{a+b}+ \frac{b^3}{b+c}+\frac{c^3}{c+a} \geq 3\frac{a^3+\lambda b^3+\gamma c^3}{2(a+b+c)}

Posté par
Sylvieg Moderateur
re : Inequation 28-08-17 à 17:32

En fait \frac{a^3}{a+b}+ \frac{b^3}{b+c}+\frac{c^3}{c+a} = (1+\frac{1}{\lambda}+\frac{1}{\gamma})(\frac{a^3+\lambda b^3+\gamma c^3}{2(a+b+c)})

Et ça coince car 1+\frac{1}{\lambda}+\frac{1}{\gamma} \le 3

Posté par
Krayz
re : Inequation 28-08-17 à 22:01

Sympa les maths de prépa !!!!

C'est plus facile en terminale S :p

Posté par
Sylvieg Moderateur
re : Inequation 29-08-17 à 07:42

Bonjour,
En fait ce sujet n'a pas grand chose à voir avec des "maths de prépa"
Bonne rentrée !

Posté par
alainpaul
re : Inequation 29-08-17 à 10:47

Bonjour,

Je tourne en rond,le type de solution que je propose soulève des difficultés.

Si l'on pose maintenant que le premier rapport : \frac{a^3}{a+b} est le plus petit alors ça ne coince plus . . .
Mais la solution ne saurait dépendre de l' hypothèse!!!

Alain

1 2 3 +




Vous devez être membre accéder à ce service...

Pas encore inscrit ?

1 compte par personne, multi-compte interdit !

Ou identifiez-vous :


Rester sur la page

Inscription gratuite

Fiches en rapport

parmi 1675 fiches de maths

Désolé, votre version d'Internet Explorer est plus que périmée ! Merci de le mettre à jour ou de télécharger Firefox ou Google Chrome pour utiliser le site. Votre ordinateur vous remerciera !